Comparing summations with integrals

In summary, the conversation discusses the inequality ##\sum_{n=1}^N \arctan{(n)} \geq N \arctan{(N)}-(1/2)\ln{(1+N^2)}## and the determination of numbers ##\alpha## and ##\beta## such that ##\lim\limits_{N\to\infty} N^\alpha \sum_{n=1}^N \arctan{(n)} =\beta##. The approach suggested is to rewrite the inequality as a Riemann sum and use the squeeze theorem to find an upper bound. Additionally, there is discussion about finding an upper bound for the upper Riemann sum and using the We
  • #1
schniefen
178
4
Homework Statement
1. Show that ##\sum_{n=1}^N \arctan{(n)} \geq N \arctan{(N)}-(1/2)\ln{(1+N^2)}##
2. Determine numbers ##\alpha##, ##\beta## such that ##\lim\limits_{N\to\infty} N^\alpha \sum_{n=1}^N \arctan{(n)} =\beta## with ##\beta \neq 0##
Relevant Equations
##\int_a^b f(x) dx= F(b)-F(a)##
1. ##\sum_{n=1}^N \arctan{(n)} \geq N \arctan{(N)}-(1/2)\ln{(1+N^2)} \iff \sum_{n=1}^N \arctan{(n)} \geq N \int_0^N \frac{1}{1+x^2} dx - \int_0^N \frac{x}{1+x^2} dx##

Where do I go from here? I've tried understanding this graphically, but to no avail.

2. Maybe this follows from finding an upper and lower bound for ##N^\alpha \sum_{n=1}^N \arctan{(n)}##, and then somehow applying the squeeze theorem?
 
Physics news on Phys.org
  • #2
You should write the right hand side as an integral of ## \operatorname{arctan}## and the left as an upper Riemann sum for this integral.
 
  • Like
Likes schniefen
  • #3
schniefen said:
Problem Statement: 1. Show that ##\sum_{n=1}^N \arctan{(n)} \geq N \arctan{(N)}-(1/2)\ln{(1+N^2)}##
2. Determine numbers ##\alpha##, ##\beta## such that ##\lim\limits_{N\to\infty} N^\alpha \sum_{n=1}^N \arctan{(n)} =\beta## with ##\beta \neq 0##
Relevant Equations: ##\int_a^b f(x) dx= F(b)-F(a)##

1. ##\sum_{n=1}^N \arctan{(n)} \geq N \arctan{(N)}-(1/2)\ln{(1+N^2)} \iff \sum_{n=1}^N \arctan{(n)} \geq N \int_0^N \frac{1}{1+x^2} dx - \int_0^N \frac{x}{1+x^2} dx##

Where do I go from here? I've tried understanding this graphically, but to no avail.
This looks to me like a Riemann sum for an integral: ##\int_{x = 0}^N \arctan(x)~dx##, or maybe ##\int_{x = 1}^{N + 1} \arctan(x)~dx##. That's the approach I would take for starters.
schniefen said:
2. Maybe this follows from finding an upper and lower bound for ##N^\alpha \sum_{n=1}^N \arctan{(n)}##, and then somehow applying the squeeze theorem?
 
  • Like
Likes schniefen
  • #4
If we have ##\sum_{n=1}^N \arctan{(n)} \geq \int_0^N \arctan{(x)} dx##, then ##N^\alpha \sum_{n=1}^N \arctan{(n)}## is simply the latter multiplied by a constant ##N^\alpha##. To obtain this expression in the first place, would it be reasonable to find an upper bound to the upper Riemann sum? Does it exist?
 
  • #5
By the way, out of curiosity, how does one get ##\int_0^N \arctan{(x)} dx## from ##N \int_0^N \frac{1}{1+x^2} dx - \int_0^N \frac{x}{1+x^2} dx##.

I get

##N \int_0^N \frac{1}{1+x^2} dx - \int_0^N \frac{x}{1+x^2} dx
\iff N\int_0^N \frac{1-x}{1+x^2} dx ##
How does one incorporate the ##N## in the integral and turn this into ##\arctan##?
 
  • #6
I do not understand your questions, sorry.
schniefen said:
To obtain this expression in the first place, would it be reasonable to find an upper bound to the upper Riemann sum? Does it exist?
What does this mean? What expression? And why do you want to find an upper bound for something which is already an upper bound for something else? Why? What for?

In order ##N^\alpha \sum_{n=1}^N \operatorname{arctan} n## to converge, the ##\alpha## has to be so small, that it compensates the ##N\cdot\frac{\pi}{2}## term of the ##\operatorname{arctan}## as well as the increasing ##N##.
schniefen said:
By the way, out of curiosity, how does one get ##\int_0^N \arctan{(x)} dx## from ##N \int_0^N \frac{1}{1+x^2} dx - \int_0^N \frac{x}{1+x^2} dx##.

I get

##N \int_0^N \frac{1}{1+x^2} dx - \int_0^N \frac{x}{1+x^2} dx
\iff N\int_0^N \frac{1-x}{1+x^2} dx ##
How does one incorporate the ##N## in the integral and turn this into ##\arctan##?
I was talking about $$\int \operatorname{arctan}\frac{x}{a}\,dx = x\operatorname{arctan}\frac{x}{a}-\frac{a}{2}\log(a^2+x^2)$$ and not about the derivative.

I'm not sure how to differentiate ##\operatorname{arctan}##. I would try the inverse function theorem or a Weierstraß substitution to integrate ##\frac{1}{1+x^2}##.
 
  • #7
Since

##N \arctan{(N)}-(1/2)\ln{(1+N^2)}=\int_0^N \arctan{(x)} dx##​

but ##N \arctan{(N)}-(1/2)\ln{(1+N^2)}## is also equal to

##N \int_0^N \frac{1}{1+x^2} dx - \int_0^N \frac{x}{1+x^2} dx = N\int_0^N \frac{1-x}{1+x^2} dx##
How does one get that ##N\int_0^N \frac{1-x}{1+x^2} dx =\int_0^N \arctan{(x)} dx## ?
 
  • #8
Why is ##\pi \,\int_0^{\pi} \frac{4x}{\pi^3}\,dx = \int_0^{\pi} \operatorname{sin}x\,dx\,?##

Finite integrals are just a real number. So why are two different descriptions of a real number equal? Of course as ##\frac{d}{dx}\operatorname{arctan}x = \frac{1}{1+x^2} ## there are connections between those expressions. My suspicion is that this will lead to the Weierstraß substitution.
 
  • #9
Okay. So ##N\int_0^N \frac{1-x}{1+x^2} dx## is not necessarily equal to ##\int_0^N \arctan{(x)} dx##?
fresh_42 said:
In order ##N^\alpha \sum_{n=1}^N \operatorname{arctan} n## to converge, the ##\alpha## has to be so small, that it compensates the ##N\cdot\frac{\pi}{2}## term of the ##\operatorname{arctan}## as well as the increasing ##N##.
Can there possibly be any ##\alpha## that compensates for this? Won't ##N^{\alpha}## yield ##[\infty]^{\alpha}## as ##N\to\infty##? If ##\alpha<0##, the limit ##\beta## will be 0 (but ##\beta\neq0##), and for all other ##\alpha## the limit will be infinite, or?
 
  • #10
schniefen said:
Okay. So ##N\int_0^N \frac{1-x}{1+x^2} dx## is not necessarily equal to ##\int_0^N \arctan{(x)} dx##?
It is equal, you can calculate both sides. But what does it mean if two formulas yield the same result? In our case there are connections between ##\operatorname{arctan}x ## and ##\frac{1}{x^2+1}## but not obvious ones. As mentioned, I think the Weierstaß substitution is a good place for further investigations.
Can there possibly be any ##\alpha## that compensates for this? Won't ##N^{\alpha}## yield ##[\infty]^{\alpha}## as ##N\to\infty##? If ##\alpha<0##, the limit ##\beta## will be 0 (but ##\beta\neq0##), and for all other ##\alpha## the limit will be infinite, or?
Write it down as your result. However, you have forgotten a case!
 

What is the difference between a summation and an integral?

A summation is a mathematical operation that involves adding a sequence of numbers together, while an integral is a mathematical operation that involves finding the area under a curve. In other words, a summation is a discrete process while an integral is a continuous process.

When should I use a summation versus an integral?

A summation is typically used when dealing with discrete data or events, such as counting the number of objects or occurrences. An integral is used when dealing with continuous data or events, such as measuring the area or volume of a shape.

How do I convert a summation into an integral?

To convert a summation into an integral, you can use the concept of Riemann sums. This involves dividing the interval into smaller subintervals and approximating the area under the curve using rectangles. As the number of subintervals increases, the approximation becomes more accurate, approaching the actual integral.

Can an integral be thought of as a continuous summation?

Yes, an integral can be thought of as a continuous summation. Just as a summation adds up discrete values, an integral adds up infinitesimally small values to find the total area under a curve. In fact, the integral is defined as the limit of a summation as the subintervals approach zero.

What are some real-world applications of comparing summations with integrals?

Summations and integrals are used extensively in various fields of science and engineering, such as physics, chemistry, and economics. For example, integrals are used to calculate the work done by a force, while summations are used to calculate the total cost of a project. They are also used in data analysis to find patterns and trends in large datasets.

Similar threads

  • Calculus and Beyond Homework Help
Replies
3
Views
987
  • Calculus and Beyond Homework Help
Replies
14
Views
2K
  • Calculus and Beyond Homework Help
Replies
12
Views
1K
  • Calculus and Beyond Homework Help
Replies
14
Views
183
  • Calculus and Beyond Homework Help
Replies
10
Views
411
  • Calculus and Beyond Homework Help
Replies
3
Views
237
  • Calculus and Beyond Homework Help
Replies
4
Views
640
  • Calculus and Beyond Homework Help
Replies
3
Views
1K
  • Calculus and Beyond Homework Help
Replies
16
Views
1K
  • Calculus and Beyond Homework Help
Replies
6
Views
743
Back
Top